メインコンテンツに移動します。
Microsoft
|
Math Solver
解
演習
遊ぶ
トピック
代数入門
平均
並数
最大公約数
最小公倍数
演算の順序
分数
混合分数
素因数分解
指数
根
代数
同類項をまとめる
変数を解く
因数
展開する
分数を求める
線形方程式
二次方程式
不等式
方程式のシステム
行列
三角法
簡約する
評価
グラフ
方程式を解く
微積分
導関数
積分
極限値
代数入力
三角法の入力
微積分入力
Matrix の入力
解
演習
遊ぶ
トピック
代数入門
平均
並数
最大公約数
最小公倍数
演算の順序
分数
混合分数
素因数分解
指数
根
代数
同類項をまとめる
変数を解く
因数
展開する
分数を求める
線形方程式
二次方程式
不等式
方程式のシステム
行列
三角法
簡約する
評価
グラフ
方程式を解く
微積分
導関数
積分
極限値
代数入力
三角法の入力
微積分入力
Matrix の入力
基本的な
代数
三角法
微積分
統計
行列
文字
計算
2^{5}\times 3^{2}\times 5
クイズ
Algebra
次に類似した 5 個の問題:
factor(1440)
Web 検索からの類似の問題
factor9.12
https://www.tiger-algebra.com/drill/factor9.12/
(912/100) Final result : 228 ——— = 9.12000 25 Reformatting the input : Changes made to your input should not affect the solution: (1): "9.12" was replaced by "(912/100)". Step by step solution : ...
findlcm.5,4,2
https://www.tiger-algebra.com/drill/findlcm.5,4,2/
Error - Decimal point not allowed here lcm(5,4,2) LCM(5,4,2) Least Common Multiple is : 20 Calculate Least Common Multiple for : 5, 4 and 2 Factorize of the ...
The number of ordered triples (a, b, c) of positive integers which satisfy the simultaneous equations ab + bc = 44, ac + bc = 33
https://math.stackexchange.com/q/664860
Your solution is correct. Noe that a=1,b-c=11 and a=11, b-c=1 both lead to a+b=12+c. Then 33=ac+bc=(12+c)c indeed has no solution. [There's an alomost-soution: c=-1 gives -33; so I wonder ...
Determinant using factor theorem
https://math.stackexchange.com/q/2660747
Your attempt is fine. And all the further explanations you need are already given by user348749 in How to solve this determinant I could just rephrase this here: Since for x+y+z=0, all three ...
Cannot find length of repeating block in decimal expansion for \frac{17}{78}
https://math.stackexchange.com/questions/802448/cannot-find-length-of-repeating-block-in-decimal-expansion-for-frac1778
Note that the period for a prime p is a factor of \varphi (p)=p-1 but need not be equal to it. This is because 10^{p-1} \equiv 1 \mod p. The period is the least n for which p|(10^n-1). If ...
Find the vertices of the polytope
https://math.stackexchange.com/questions/295533/find-the-vertices-of-the-polytope
Write the first constraint as \sum_j a_{1j} = m, with m < n ( I am just replacing the variable x by m). Since a_{i1}+a_{i2} = 1, and a_{ij}\geq 0, we see that this is equivalent to a_{i2} = 1-a_{i1} ...
その他の 項目
共有
コピー
クリップボードにコピー済み
類似問題
factor(100)
factor(42)
factor(662)
factor(330)
factor(1440)
factor(7700)
トップに戻る